Difference between revisions of "2006 USAMO Problems/Problem 5"

Line 1: Line 1:
 
== Problem ==
 
== Problem ==
  
A mathematical frog jumps along the number line. The frog starts at 1, and jumps according to the following rule: if the frog is at integer <math> \displaystyle n </math>, then it can jump either to <math> \displaystyle n+1 </math> or to <math>n+2^{m_n+1}</math> where <math>2^{m_n}</math> is the largest power of 2 that is a factor of <math> \displaystyle n </math>. Show that if <math>k\ge 2</math> is a positive integer and <math> \displaystyle i </math> is a nonnegative integer, then the minimum number of jumps needed to reach <math> \displaystyle 2^i k </math> is greater than the minimum number of jumps needed to reach <math> \displaystyle 2^i </math>.
+
A mathematical frog jumps along the number line. The frog starts at 1, and jumps according to the following rule: if the frog is at integer <math>n </math>, then it can jump either to <math>n+1 </math> or to <math>n+2^{m_n+1}</math> where <math>2^{m_n}</math> is the largest power of 2 that is a factor of <math>n </math>. Show that if <math>k\ge 2</math> is a positive integer and <math>i </math> is a nonnegative integer, then the minimum number of jumps needed to reach <math>2^i k </math> is greater than the minimum number of jumps needed to reach <math>2^i </math>.
  
 
== Solution ==
 
== Solution ==
Line 14: Line 14:
  
 
[[Category:Olympiad Number Theory Problems]]
 
[[Category:Olympiad Number Theory Problems]]
 +
{{MAA Notice}}

Revision as of 13:41, 4 July 2013

Problem

A mathematical frog jumps along the number line. The frog starts at 1, and jumps according to the following rule: if the frog is at integer $n$, then it can jump either to $n+1$ or to $n+2^{m_n+1}$ where $2^{m_n}$ is the largest power of 2 that is a factor of $n$. Show that if $k\ge 2$ is a positive integer and $i$ is a nonnegative integer, then the minimum number of jumps needed to reach $2^i k$ is greater than the minimum number of jumps needed to reach $2^i$.

Solution

This problem needs a solution. If you have a solution for it, please help us out by adding it.

See Also

The problems on this page are copyrighted by the Mathematical Association of America's American Mathematics Competitions. AMC logo.png